Bạn chưa đăng nhập. Vui lòng đăng nhập để hỏi bài

Những câu hỏi liên quan
Tiến Nguyễn Minh
Xem chi tiết
Vũ Tiến Manh
21 tháng 10 2019 lúc 22:19

1) Áp dụng bunhiacopxki ta được \(\sqrt{\left(2a^2+b^2\right)\left(2a^2+c^2\right)}\ge\sqrt{\left(2a^2+bc\right)^2}=2a^2+bc\), tương tự với các mẫu ta được vế trái \(\le\frac{a^2}{2a^2+bc}+\frac{b^2}{2b^2+ac}+\frac{c^2}{2c^2+ab}\le1< =>\)\(1-\frac{bc}{2a^2+bc}+1-\frac{ac}{2b^2+ac}+1-\frac{ab}{2c^2+ab}\le2< =>\)

\(\frac{bc}{2a^2+bc}+\frac{ac}{2b^2+ac}+\frac{ab}{2c^2+ab}\ge1\)<=> \(\frac{b^2c^2}{2a^2bc+b^2c^2}+\frac{a^2c^2}{2b^2ac+a^2c^2}+\frac{a^2b^2}{2c^2ab+a^2b^2}\ge1\)  (1) 

áp dụng (x2 +y2 +z2)(m2+n2+p2\(\ge\left(xm+yn+zp\right)^2\)

(2a2bc +b2c2 + 2b2ac+a2c2 + 2c2ab+a2b2). VT\(\ge\left(bc+ca+ab\right)^2\)   <=> (ab+bc+ca)2. VT \(\ge\left(ab+bc+ca\right)^2< =>VT\ge1\)  ( vậy (1) đúng)

dấu '=' khi a=b=c

Khách vãng lai đã xóa
HD Film
21 tháng 10 2019 lúc 22:26

4b, \(\frac{a^3}{a^2+b^2}+\frac{b^3}{b^2+c^2}+\frac{c^3}{c^2+a^2}=1-\frac{ab^2}{a^2+b^2}+1-\frac{bc^2}{b^2+c^2}+1-\frac{ca^2}{a^2+c^2}\)

\(\ge3-\frac{ab^2}{2ab}-\frac{bc^2}{2bc}-\frac{ca^2}{2ac}=3-\frac{\left(a+b+c\right)}{2}=\frac{3}{2}\)

Khách vãng lai đã xóa
HD Film
21 tháng 10 2019 lúc 22:35

4c, 

\(\frac{a+1}{b^2+1}+\frac{b+1}{c^2+1}+\frac{c+1}{a^2+1}=a+b+c-\frac{b^2}{b^2+1}-\frac{c^2}{c^2+1}-\frac{a^2}{a^2+1}+3--\frac{b^2}{b^2+1}-\frac{c^2}{c^2+1}-\frac{a^2}{a^2+1}\)\(\ge6-2\cdot\frac{\left(a+b+c\right)}{2}=3\)

Khách vãng lai đã xóa
Lê Minh Đức
Xem chi tiết
Akai Haruma
9 tháng 10 2017 lúc 17:28

Ta có \(a^2b^2+b^2c^2+c^2a^2\geq a^2b^2c^2\Leftrightarrow \frac{1}{a^2}+\frac{1}{b^2}+\frac{1}{c^2}\geq 1\)

BĐT cần chứng minh tương đương với \(\frac{\frac{1}{c^3}}{\frac{1}{a^2}+\frac{1}{b^2}}+\frac{\frac{1}{b^3}}{\frac{1}{a^2}+\frac{1}{c^2}}+\frac{\frac{1}{a^3}}{\frac{1}{b^2}+\frac{1}{c^2}}\geq \frac{\sqrt{3}}{2}\)

Đặt \((\frac{1}{a},\frac{1}{b},\frac{1}{c})=(x,y,z)\). Bài toán trở thành: 

Cho \(x,y,z>0|x^2+y^2+z^2\geq 1\). CMR \(P=\frac{x^3}{y^2+z^2}+\frac{y^3}{z^2+x^2}+\frac{z^3}{x^2+y^2}\geq \frac{\sqrt{3}}{2}\)

Lời giải:

 Áp dụng BĐT Cauchy -Schwarz:

\(P=\frac{x^4}{xy^2+xz^2}+\frac{y^4}{yz^2+yx^2}+\frac{z^4}{zx^2+zy^2}\geq \frac{(x^2+y^2+^2)^2}{x^2(y+z)+y^2(x+z)+z^2(x+y)}\) (1)

Không mất tính tổng quát, giả sử \(x\geq y\geq z\Rightarrow x^2\geq y^2\geq z^2\) 

Và \(y+z\leq z+x\leq x+y\). Khi đó, áp dụng BĐT Chebyshev: 

\(3[x^2(y+z)+y^2(x+z)+z^2(x+y)]\leq (x^2+y^2+z^2)(y+z+x+z+x+y)\)

\(\Leftrightarrow x^2(y+z)+y^2(x+z)+z^2(x+y)\leq \frac{2(x^2+y^2+z^2)(x+y+z)}{3}\)

Theo hệ quả của BĐT Am-Gm thì: \((x+y+z)^2\leq 3(x^2+y^2+z^2)\Rightarrow x+y+z\leq \sqrt{3(x^2+y^2+z^2)}\)

\(\Rightarrow x^2(y+z)+y^2(x+z)+z^2(x+y)\leq \frac{2(x^2+y^2+z^2)\sqrt{3(x^2+y^2+z^2)}}{3}\) (2)

Từ (1),(2) suy ra \(P\geq \frac{3(x^2+y^2+z^2)^2}{2(x^2+y^2+z^2)\sqrt{3(x^2+y^2+z^2)}}=\frac{\sqrt{3(x^2+y^2+z^2)}}{2}\geq \frac{\sqrt{3}}{2}\)

Ta có đpcm

Dáu bằng xảy ra khi \(x=y=z=\frac{1}{\sqrt{3}}\Leftrightarrow a=b=c=\sqrt{3}\)

Kiệt Nguyễn
5 tháng 5 2020 lúc 12:58

Đặt \(x=\frac{1}{a};y=\frac{1}{b};z=\frac{1}{c}\)

Khi đó giả thiết được viết lại là \(x^2+y^2+z^2\ge1\)và ta cần chứng minh \(\frac{x^3}{y^2+z^2}+\frac{y^3}{z^2+x^2}+\frac{z^3}{x^2+y^2}\ge\frac{\sqrt{3}}{2}\)(*)

Áp dụng BĐT Bunhiacopxki dạng phân thức, ta được:

\(VT_{\left(^∗\right)}=\frac{x^4}{x\left(y^2+z^2\right)}+\frac{y^4}{y\left(z^2+x^2\right)}+\frac{z^4}{z\left(x^2+y^2\right)}\)\(\ge\frac{\left(x^2+y^2+z^2\right)^2}{x\left(y^2+z^2\right)+y\left(z^2+x^2\right)+z\left(x^2+y^2\right)}\)

Đến đây ta đi chứng minh \(\frac{\left(x^2+y^2+z^2\right)^2}{x\left(y^2+z^2\right)+y\left(z^2+x^2\right)+z\left(x^2+y^2\right)}\ge\frac{\sqrt{3}}{2}\)

\(\Leftrightarrow2\left(x^2+y^2+z^2\right)^2\)\(\ge\sqrt{3}\left[x\left(y^2+z^2\right)+y\left(z^2+x^2\right)+z\left(x^2+y^2\right)\right]\)

Ta có: \(x\left(y^2+z^2\right)=\frac{1}{\sqrt{2}}\sqrt{2x^2\left(y^2+z^2\right)\left(y^2+z^2\right)}\)\(\le\frac{1}{\sqrt{2}}\sqrt{\left(\frac{2x^2+y^2+z^2+y^2+z^2}{3}\right)^3}\)

\(=\frac{2\sqrt{3}}{9}\left(x^2+y^2+z^2\right)\sqrt{x^2+y^2+z^2}\)

Tương tự ta có: \(y\left(z^2+x^2\right)\le\frac{2\sqrt{3}}{9}\left(x^2+y^2+z^2\right)\sqrt{x^2+y^2+z^2}\)

\(z\left(x^2+y^2\right)\le\frac{2\sqrt{3}}{9}\left(x^2+y^2+z^2\right)\sqrt{x^2+y^2+z^2}\)

Cộng theo vế của 3 BĐT trên, ta được: 

\(\text{∑}_{cyc}\left[x\left(y^2+z^2\right)\right]\le\frac{2\sqrt{3}}{3}\left(x^2+y^2+z^2\right)\sqrt{x^2+y^2+z^2}\)

\(\Leftrightarrow\sqrt{3}\text{∑}_{cyc}\left[x\left(y^2+z^2\right)\right]\le2\left(x^2+y^2+z^2\right)\sqrt{x^2+y^2+z^2}\)

Cuối cùng ta cần chứng minh được

\(2\left(x^2+y^2+z^2\right)\sqrt{x^2+y^2+z^2}\le2\left(x^2+y^2+z^2\right)^2\)

\(\Leftrightarrow x^2+y^2+z^2\ge1\)(đúng)

Đẳng thức xảy ra khi \(x=y=z=\frac{1}{\sqrt{3}}\Rightarrow a=b=c=\sqrt{3}\)

Khách vãng lai đã xóa
Nghiêm Thị Nhân Đức
Xem chi tiết
Copxki Minh
2 tháng 12 2020 lúc 22:25

Đặt \(\left(\frac{1}{a},\frac{1}{b},\frac{1}{c}\right)=\left(x,y,z\right)\)

\(x+y+z\ge\frac{x^2+2xy}{2x+y}+\frac{y^2+2yz}{2y+z}+\frac{z^2+2zx}{2z+x}\)

\(\Leftrightarrow x+y+z\ge\frac{3xy}{2x+y}+\frac{3yz}{2y+z}+\frac{3zx}{2z+x}\)

\(\frac{3xy}{2x+y}\le\frac{3}{9}xy\left(\frac{1}{x}+\frac{1}{x}+\frac{1}{y}\right)=\frac{1}{3}\left(x+2y\right)\)

\(\Rightarrow\Sigma_{cyc}\frac{3xy}{2x+y}\le\frac{1}{3}\left[\left(x+2y\right)+\left(y+2z\right)+\left(z+2x\right)\right]=x+y+z\)

Dấu "=" xảy ra khi x=y=z

Khách vãng lai đã xóa
Phạm Thị Thu Uyên
Xem chi tiết
Thắng Nguyễn
11 tháng 1 2018 lúc 21:59

Áp dụng BĐT Cauchy-Schwarz ta có:

\(\sqrt{a\left(3a+b\right)}+\sqrt{b\left(3b+a\right)}\)

\(\le\sqrt{\left(a+b\right)\left(3a+b+3b+a\right)}\)

\(=\sqrt{4\left(a+b\right)^2}=2\left(a+b\right)\)

\(\Rightarrow\frac{a+b}{\sqrt{a\left(3a+b\right)}+\sqrt{b\left(3b+a\right)}}\ge\frac{a+b}{2\left(a+b\right)}=\frac{1}{2}\)

KAl(SO4)2·12H2O
28 tháng 5 2018 lúc 20:31

Áp dụng Cauchy-Schwarz ta có:

\(\frac{a+b}{\sqrt{a\left(3a+b\right)}+\sqrt{b\left(3b+a\right)}}=\frac{1}{2}\)

Tiến Nguyễn Minh
Xem chi tiết
Copxki Minh
23 tháng 11 2020 lúc 23:52

1)

Ta có: \(M=\Sigma_{cyc}\frac{\sqrt{3}\left(a+b+4c\right)}{\sqrt{3\left(a+b\right)\left(a+b+4c\right)}}\ge\Sigma_{cyc}\frac{\sqrt{3}\left(a+b+4c\right)}{\frac{3\left(a+b\right)+\left(a+b+4c\right)}{2}}=\Sigma_{cyc}\frac{\sqrt{3}\left(a+b+4c\right)}{2\left(a+b+c\right)}=3\sqrt{3}\)

Dấu "=" xảy ra khi a=b=c

Khách vãng lai đã xóa
Copxki Minh
24 tháng 11 2020 lúc 9:53

2)

\(\Sigma_{cyc}\sqrt[3]{\left(\frac{2a}{ab+1}\right)^2}=\Sigma_{cyc}\frac{2a}{\sqrt[3]{2a\left(ab+1\right)^2}}\ge\Sigma_{cyc}\frac{2a}{\frac{2a+\left(ab+1\right)+\left(ab+1\right)}{3}}=3\Sigma_{cyc}\frac{a}{ab+a+1}\)

Ta có bổ đề: \(\frac{a}{ab+a+1}+\frac{b}{bc+b+1}+\frac{c}{ca+c+1}=1\left(abc=1\right)\)

\(\Rightarrow\Sigma_{cyc}\sqrt[3]{\left(\frac{2a}{ab+1}\right)^2}\ge3\)

Khách vãng lai đã xóa
Vân Phi Tuyết
Xem chi tiết
alibaba nguyễn
31 tháng 3 2017 lúc 19:00

Ta có: 

\(\frac{a+b}{\sqrt{a\left(3a+b\right)}+\sqrt{b\left(3b+a\right)}}=\frac{2\left(a+b\right)}{\sqrt{4a\left(3a+b\right)}+\sqrt{4b\left(3b+a\right)}}\)

\(\ge\frac{2\left(a+b\right)}{\frac{4a+3a+b}{2}+\frac{4b+3b+a}{2}}=\frac{2\left(a+b\right)}{4\left(a+b\right)}=\frac{1}{2}\)

Dấu = xảy ra khi \(a=b\)

Thắng Nguyễn
31 tháng 3 2017 lúc 21:40

Áp dụng BĐT Cauchy-Schwarz ta có: 

\(\sqrt{a\left(3a+b\right)}+\sqrt{b\left(3b+a\right)}=\sqrt{a}\sqrt{3a+b}+\sqrt{b}\sqrt{3b+a}\)

\(\le\sqrt{\left(a+b\right)\left(3a+b+3b+a\right)}=2\left(a+b\right)\)

\(\Rightarrow\frac{a+b}{\sqrt{a\left(3a+b\right)}+\sqrt{b\left(3b+a\right)}}\ge\frac{a+b}{2\left(a+b\right)}=\frac{1}{2}\)

Đẳng thức xảy ra khi \(a=b\)

trương thế bách
2 tháng 4 2017 lúc 16:12

em mới học lớp 6 thôi,bài này đối với em quá khó ,mong chị thông cảm và chúc chị học giỏi

Admin (a@olm.vn)
Xem chi tiết
Lê Trường Lân
Xem chi tiết
Tran Le Khanh Linh
16 tháng 5 2020 lúc 21:06

Bài 1: diendantoanhoc.net

Đặt \(a=\frac{1}{x};b=\frac{1}{y};c=\frac{1}{z}\) BĐT cần chứng minh trở thành

\(\frac{x}{\sqrt{3zx+2yz}}+\frac{x}{\sqrt{3xy+2xz}}+\frac{x}{\sqrt{3yz+2xy}}\ge\frac{3}{\sqrt{5}}\)

\(\Leftrightarrow\frac{x}{\sqrt{5z}\cdot\sqrt{3x+2y}}+\frac{y}{\sqrt{5x}\cdot\sqrt{3y+2z}}+\frac{z}{\sqrt{5y}\cdot\sqrt{3z+2x}}\ge\frac{3}{5}\)

Theo BĐT AM-GM và Cauchy-Schwarz ta có:

\( {\displaystyle \displaystyle \sum }\)\(_{cyc}\frac{x}{\sqrt{5z}\cdot\sqrt{3x+2y}}\ge2\)\( {\displaystyle \displaystyle \sum }\)\(\frac{x}{3x+2y+5z}\ge\frac{2\left(x+y+z\right)^2}{x\left(3x+2y+5z\right)+y\left(5x+3y+2z\right)+z\left(2x+5y+3z\right)}\)

\(=\frac{2\left(x+y+z\right)^2}{3\left(x^2+y^2+z^2\right)+7\left(xy+yz+zx\right)}\)

\(=\frac{2\left(x+y+z\right)^2}{3\left(x^2+y^2+z^2\right)+\frac{1}{3}\left(xy+yz+zx\right)+\frac{20}{3}\left(xy+yz+zx\right)}\)

\(\ge\frac{2\left(x+y+z\right)^2}{3\left(x^2+y^2+z^2\right)+\frac{1}{3}\left(x^2+y^2+z^2\right)+\frac{20}{3}\left(xy+yz+zx\right)}\)

\(=\frac{2\left(x^2+y^2+z^2\right)}{5\left[x^2+y^2+z^2+2\left(xy+yz+zx\right)\right]}=\frac{3}{5}\)

Khách vãng lai đã xóa
Tran Le Khanh Linh
16 tháng 5 2020 lúc 21:07

Bổ sung bài 1:

BĐT được chứng minh

Đẳng thức xảy ra <=> a=b=c

Khách vãng lai đã xóa
Đỗ Thu Hà
Xem chi tiết